ChaseDream
搜索
返回列表 发新帖
本题详情

本贴相关题目 OG (OZGC)

00:00:00

The program to control the entry of illegal drugs into the country was a failure in 1987. If the program had been successful, the wholesale price of most illegal drugs would not have dropped substantially in 1987.

The argument in the passage depends on which of the following assumptions?

正确答案: E

更多相关帖子

524

帖子

15

好友

4712

积分

ChaseDream

注册时间
2003-03-17
精华
8
解析
查看: 2478|回复: 4
打印 上一主题 下一主题

OG-80 (llinkage wrong)

[复制链接]
楼主
发表于 2005-6-17 09:03:00 | 只看该作者

OG-80 (llinkage wrong)

80.The program to control the entry of illegal drugs into the country was a failure in 1987. If the program had been successful, the wholesale price of most illegal drugs would not have dropped substantially in 1987.



The argument in the passage depends on which of the following assumptions?


(A) The supply of illegal drugs dropped substantially in 1987.


(B) The price paid for most illegal drugs by the average consumer did not drop substantially in 1987.


(C) Domestic production of illegal drugs increased at a higher rate than did the entry of such drugs into the country.


(D) The wholesale price of a few illegal drugs increased substantially in 1987. E


(E) A drop in demand for most illegal drugs in 1987 was not the sole cause of the drop in their wholesale price.



80.


The only choice that must be true in order to conclude legitimately from the drop in the wholesale price of illegal drugs that the program was a failure is choice E, the best answer. If the drop in price was caused by a drop in demand, there is no reason to suspect that there has been any increase in supply caused by drugs entering the country.



The other choices can be false without affecting the argument. The supply of illegal drugs need not have dropped (choice A), and the retail price could have dropped (choice B). The entry of illegal drugs could have risen at a higher rate than domestic production (choice C), and no illegal drug need have undergone a substantial price rise (choice D).


看OG关于E的解释怎么样也看不出个所以然


还有A为何不正确?

沙发
发表于 2005-6-17 15:31:00 | 只看该作者

价格会下降影响的因素一个是供给增多(管制失败导致),另一个是需求增加。


所以a恰好是反的。


如果不是承认了还有其他的因素引起了价格的下跌,那么那个管制成功与否与价格都是没有关系的


not+weaken

板凳
发表于 2005-6-20 15:49:00 | 只看该作者

OG的解释是用原文结论的逆否命题来说的(不知道为什么ETS非要用逆否命题来解释),原文结论是“计划成果 推出 价格不下降”, 解释中说The only choice that must be true in order to conclude legitimately from the drop in the wholesale price of illegal drugs that the program was a failure ,说的是“价格下降推出计划失败” 其实和原文是一回事。 个人觉得这题没必要用逆否命题。


此题的解体思路是,原文结论为条件结果,对结论的假设就是,排除充分条件出现,结果不出现的情况,排除其他因数会导致结果不出现,或者是条件一定能推出结果 (lawyer NN 在他的文章中讲过,建议你仔细体会一下lawyer的文章),而E就是在排除“在计划成果的情况下,demand的下降使价格下降”

地板
发表于 2005-6-24 18:11:00 | 只看该作者

there are 2 main reasons cause the wholesale price deline : the drop of demand or the increasing of supply.


the anwser E said:A drop in demand for most illegal drugs in 1987 was not the sole cause of the drop in their wholesale price


from this assumption, the conclusion can be inferred is: the increasing of supply is the sole cause.which conclude that the contral program is failure on contral.


2楼的DX有个笔误,是需求减少,而不是增加。如果需求增加会导致价格升高。

5#
发表于 2005-7-2 06:31:00 | 只看该作者

from this assumption, the conclusion can be inferred is: the increasing of supply is the sole cause.which conclude that the contral program is failure on contral.


请问楼上的朋友,这个结论如何得出。。

您需要登录后才可以回帖 登录 | 立即注册

Mark一下! 看一下! 顶楼主! 感谢分享! 快速回复:

手机版|ChaseDream|GMT+8, 2024-9-19 11:14
京公网安备11010202008513号 京ICP证101109号 京ICP备12012021号

ChaseDream 论坛

© 2003-2023 ChaseDream.com. All Rights Reserved.

返回顶部